Where did the blue box vector come from in the vector multiplication problem?

Click For Summary
The discussion centers on the origin of the blue box vector in a vector multiplication problem related to the RAO equation. The user is confused about how the blue box vector is derived from the known equation RAO = b(Sinβj + Cosβk). A suggestion is made that the blue box vector likely results from calculating the cross product of ω and r_AO, adjusted by a scalar factor of b. It is also noted that a scalar factor appears to be missing in the transition between lines of the problem. Clarification and double-checking of the calculations are recommended for accuracy.
influx
Messages
162
Reaction score
1

Homework Statement


b98fa7.png


Homework Equations



RAO = b(Sinβj+Cosβk)

The Attempt at a Solution


[/B]
The box in red is ω. However I am unsure of where they got the box in blue from? As mentioned above, RAO = b(Sinβj+Cosβk) so not sure where they got the box in blue from? I know of the vector triple product: A x (B x C) = (A•C)B - (A.B)C, but this isn't what they've done?

Thanks
 
Physics news on Phys.org
influx said:

Homework Statement


b98fa7.png


Homework Equations



RAO = b(Sinβj+Cosβk)

The Attempt at a Solution


[/B]
The box in red is ω. However I am unsure of where they got the box in blue from? As mentioned above, RAO = b(Sinβj+Cosβk) so not sure where they got the box in blue from? I know of the vector triple product: A x (B x C) = (A•C)B - (A.B)C, but this isn't what they've done?

Thanks

I would assume that they are doing the obvious thing, ie. calculating \mathbf{\omega} \times \mathbf{r}_{AO} first, and given the presence of a scalar factor of b (which goes missing in the second line before reappearing in the third) the vector in the blue box must therefore be \frac{\mathbf{\omega} \times \mathbf{r}_{AO}}{b}. (Although given that something did go missing partway through I would recommend double-checking this.)
 
Last edited:
Question: A clock's minute hand has length 4 and its hour hand has length 3. What is the distance between the tips at the moment when it is increasing most rapidly?(Putnam Exam Question) Answer: Making assumption that both the hands moves at constant angular velocities, the answer is ## \sqrt{7} .## But don't you think this assumption is somewhat doubtful and wrong?

Similar threads

  • · Replies 9 ·
Replies
9
Views
2K
  • · Replies 3 ·
Replies
3
Views
3K
  • · Replies 8 ·
Replies
8
Views
3K
  • · Replies 6 ·
Replies
6
Views
2K
  • · Replies 16 ·
Replies
16
Views
6K
  • · Replies 5 ·
Replies
5
Views
2K
  • · Replies 146 ·
5
Replies
146
Views
10K
  • · Replies 8 ·
Replies
8
Views
3K
  • · Replies 15 ·
Replies
15
Views
2K
  • · Replies 6 ·
Replies
6
Views
11K